Physiology > EXAM REVIEW > NUR 2212, PHSYCHOLOGY EAQ** | Answers Correctly Marked (100 % CORRECT) (All)

NUR 2212, PHSYCHOLOGY EAQ** | Answers Correctly Marked (100 % CORRECT)

Document Content and Description Below

PSYCH EAQ A team approach is used to help a 6-year-old boy with attention deficit-hyperactivity disorder (ADHD). What behaviors indicate that the interventions have been effective? Select all tha... t apply. 1 2 Correct4 Correct5 Is able to wait his turn when in line with others A nurse is assigned to lead a relaxation group. Which techniques should the nurse incorporate? Select all that apply. Meditation Mental imagery 3 Token economy 4 Operant conditioning Correct5 Deep-breathing exercises How should a nurse characterize a sudden terrorist act that causes the deaths of thousands of adults and children and negatively affects their families, friends, communities, and the nation? 1 2 Correct4 Adventitious A nurse has just completed a mental status examination on a newly admitted psychiatric client and returns to the nurses’ station to document the results. The nurse reflects on the client’s drawn-out explanation of the reason for the admission and concludes that excessive detail was given before the client eventually answered the questions. What mental process does the nurse identify? 1 Flight of ideas Circumstantiality When talking with a client who has alcoholism, the nurse notes that the client becomes irritable, makes excuses, and blames family and friends for the drinking problem. Which defense mechanisms does the nurse conclude that the client is using? Select all that apply. Correct1 Projection Suppression 3 Sublimation 4 Correct5 Rationalization The registered nurse managing the care of four clients is determining individual priorities. Place the following clients in order of priority, with 1 as the highest priority. A newly admitted client with schizophrenia has a treatment plan that includes participation in a physical activity group for several days before assignment to an analytical group. The basis for this decision is that the client will do what? Develop skills in managing leisure time 2 Have time to develop insight into personal problems 3 Be too disruptive to benefit from group therapy at this time Correct4 Cultivate trust before moving into a potentially anxiety-producing group The mother of an 18-year-old man comes to the local mental health center. She is extremely upset because of her son’s behavior since the young man returned from his freshman year at college. He takes his brother’s clothing, comes in at all hours, and refuses to get a job. Sometimes he is happy and outgoing, but at other times he is withdrawn. The mother asks why her son is like this. While contemplating this situation, the nurse considers that which adjectives usually apply to adolescents? 1 Anxious and unhappy 2 A young adult is being treated in the emergency department for injuries sustained as a result of physical battering by her partner. On learning that there is a history of such abuse, the nurse plans which tertiary nursing interventions? Select all that apply. 2 Correct4 Correct5 Providing her with information regarding local domestic violence shelters Certain questions are applicable in determining nursing negligence. Select all that apply. Correct1 "Was reasonable care provided?" Correct2 "Was there a breach of nursing duty?" "Was there an act of omission that resulted in harm?" "Except for the nurse’s action, would the injury have occurred?" The parents of a toddler with recently diagnosed moderate cognitive impairment state, "Our child should be able to attend college with help and medication." What should the nurse conclude? 1 A nurse is caring for a 20-year-old client. According to Erikson’s developmental psychosocial theory, what is expected by 20 years of age? 1 Correct3 Having a coherent sense of self and plans for self-actualization A small fire has been set in the dayroom garbage can by a client who is currently demonstrating manic behavior. Place the following nursing interventions in the appropriate order to best ensure client and milieu safety. After an electroconvulsive therapy treatment, a client complains of loss of memory. What is the nurse's best response? Correct1 "This is temporary; your memory will return after the therapy is done." A nurse facilitating a support group of widows and widowers recalls that research indicates that the probability of a spouse having a pathological or morbid grief response will be greater in what case? The couple had an ambivalent relationship. Correct2 The cause of the spouse's death was suicide. Based on the following documentation, to best address the client's needs, what is the nurse's initial intervention? Correct2 Offering the client a high-protein drink every hour in a quiet area of the unit A client who had to be cut out of a car after a motor vehicle collision has no visible physical effects from the ordeal. The client responds to the emergency department nurse's questions factually in a soft voice with a composed manner. What may this behavior indicate about the client? Correct1 The client is controlling the expression of feelings. An older woman comes to the mental health clinic and reports, "I've not been feeling right and haven't been able to sleep or eat since my husband died 8 months ago." The nurse determines that the client is experiencing grief associated with the loss of the husband. What supports this conclusion? 1 Inability to talk about her loss 2 Difficulty in expressing her loss Lack of sleep and the presence of symptoms of depression A primary nurse notes that a client has become jaundiced after 2 weeks of antipsychotic drug therapy. The nurse continues to administer the antipsychotic until the primary healthcare provider can be consulted. What does the nurse manager conclude about this situation? Correct1 Jaundice is sufficient reason to discontinue the antipsychotic. Which individual is coping with issues concerning dependence versus independence? 1 Infant Toddler A nurse is teaching a group of recently hired staff members about defense mechanisms. An example given is Scarlett O'Hara, in the movie Gone with the Wind, who said, "I'll think about that tomorrow." What defense mechanism does this statement reflect? 2 Splitting 3 Repression A client's hands are raw and bloody from a ritual involving frequent hand washing. Which defense mechanism does the nurse identify? Correct1 Undoing A nurse working in a mental health clinic has a caseload composed of a number of individuals and families. Which members of the caseload are at the greatest risk for mental health problems? A family with a new baby, a divorced man, and a recently retired older woman A parent of four is remanded to the psychiatric unit by the court for observation. The client was arrested and charged with abusing a 2-year-old child, who is in the pediatric intensive care unit in critical condition. The nurse approaches the client for the first time. How should the nurse anticipate that the client will likely respond? Select all that apply. By denying beating the child Correct2 By avoiding talking about the situation By asking where the other three children are 4 Correct5 By exhibiting an emotional response that is inconsistent with the degree of injury According to Erikson, a person’s adjustment to the period of senescence will depend largely on the adjustment the individual made to which earlier developmental stage? 1 Trust versus mistrust Industry versus inferiority 3 Identity versus role confusion Correct4 Generativity versus stagnation During the eighth session of a therapy group, a member who talks frequently is interrupted by one who doesn't. When the interrupting person is finished talking, the one who usually contributes says, "I'm so glad that you feel like talking today." While saying this, the client sits rigidly and looks angry. How should the nurse respond? Comment on the interrupted client's angry behavior and pleasant words. A practitioner prescribes routine checks of the client's lithium level to be performed. How many hours after the last dose of lithium should the nurse plan to obtain the blood specimen? 2 to 4 2 4 to 6 3 6 to 8 Correct4 A psychiatric unit uses a behavioral approach to determine a client's level of privileges. Which factor should a nurse use to determine whether an increase in privileges is warranted? 1 Which client in a psychiatric unit needs immediate therapeutic intervention from the nurse? A 25-year-old man who is mimicking the use of a machine gun in front of the nurse’s station 2 Correct3 A client who was sexually assaulted and is aware of the possible legal implications decides to seek prosecution of the rapist. The nurse carefully listens and documents all assessments. This is done because with a charge of rape the burden of proof has which implication? The burden of proof rests with the health team. 2 It is on the defendant to prove innocence. 3 Burden of proof must be established before the case will be heard. Correct4 The burden of proof rests with the criminal justice system in collaboration with the victim. A 17-year-old client is diagnosed with leukemia. Which statements by the teenager reflect Piaget’s cognitive processes associated with adolescence? Select all that apply. 1 Correct3 Correct5 A young woman is brought to the emergency department by friends after being sexually assaulted. The client has a small but deep laceration on her chin, as well as contusions on her arms and legs. The client appears withdrawn but calm. Place the following nursing interventions in the appropriate order to best address the client’s immediate needs. What action should the nurse manager take when it becomes apparent that communication between the nurse and the client is consistently superficial? 1 Assessing the client’s ability to understand the nurse’s questions A female adolescent in group therapy tells the other group members that while out on a pass she used marijuana because her boyfriend made her smoke it. What defense mechanism is the client using? 1 Correct3 Projection An older adult with a chronic degenerative disease progresses to the stage at which self-care is no longer possible, and admission to a long-term care facility becomes necessary. What is the major developmental conflict for this client, according to Erikson? Intimacy versus isolation Correct2 Ego integrity versus despair The nurse’s role in maintaining or promoting the health of the older adult should be based on which principle? 1 Some physiologic changes that occur as a result of aging are reversible. Thoughts of impending death are common and depressing to most older adults. 3 Older adults can better accept the dependent state that chronic illness often causes. Correct4 There is a strong correlation between successful retirement and maintaining health. A client with bipolar disorder has been admitted for alcohol detoxification, and laboratory tests are performed. Which results should prompt the nurse to notify the admitting health care provider? Select all that apply. 1 Correct3 5 A nurse begins terminating the consistent one-to-one relationship with a client who is soon to be discharged. How might the nurse expect the client to respond to the termination of their relationship? The home care nurse visits a child in whom failure to thrive has been diagnosed and makes observations that lead the nurse to suspect that the house is being used as a methamphetamine laboratory. Which observations support this conclusion? Select all that apply. Correct2 4 Correct5 Jars containing bright-yellow crystals The parents of an autistic child begin family therapy with a nurse therapist. The father states that the family members wish to share their religious beliefs with the therapist. What should the nurse do? Limit the father's discussion of religion. 2 Include the mutual discussion of religious beliefs. 3 Invite the family's religious leader to a therapy session. Correct4 Encourage family discussion of their religion in the sessions. What is the best initial approach to take with a self-accusatory, guilt-ridden client? 1 Contradict the client's persecutory delusions. A nurse in the mental health clinic is counseling a client with the diagnosis of depression. During the counseling session the client says, "Things always seem the same. They never change." The nurse suspects that the client is feeling hopeless. For what indication of hopelessness should the nurse assess the client? Correct1 Outbursts of ange An 8-year-old child is found to have oppositional defiant disorder. What behavior noted by the nurse supports this diagnosis? Correct2 Argues with adults Which client assessment does the nurse determine is inconsistent with the diagnosis of anorexia nervosa, restricting type? Engages in episodes of purging What should the nurse teach parents about childhood depression? Correct1 May appear as acting-out behavior The nurse is interviewing a female adolescent with anorexia nervosa who is malnourished and severely underweight. Which statement leads the nurse to conclude that the client is experiencing secondary gains from her behavior? 1 "I'm as big as a house." 2 "I get straight A's in school." "My mother keeps trying to get me to eat." A client with a long history of alcohol abuse who has been hospitalized for 1 week tells the nurse, "I feel much better and probably won't need any more treatment." What does the nurse conclude when evaluating the client's progress? 1 The client has accepted the illness and now must use willpower to resist alcohol. Correct3 A nurse in a mental health unit of the emergency department of a hospital frequently cares for adolescents who attempt suicide. What is important for the nurse to remember about adolescent suicide behavior? 1 2 Correct4 Boys are more likely to use lethal methods than are girls. A 42-year-old adult with a long history of alcohol abuse seeks help in one of the local hospitals. What does the nurse consider to be the major underlying factor for success in a client’s alcohol treatment program? 1 Family Correct2 Motivation A nurse is working with clients with a variety of eating disorders. Which characteristic unique to bulimia nervosa differentiates this disorder from anorexia nervosa? 1 The client is obese and attempting to lose weight. Correct2 The client behaves appropriately and looks normal. A nurse is discussing plans with a client who has decided to withdraw from alcohol. What should the nurse recommend as one of the mosteffective treatments for alcoholism? Daily administration of disulfiram 2 3 Correct4 Active membership in Alcoholics Anonymous A severely depressed male client responds to therapy and with the help of the staff begins to set some daily objectives. Which behavior mostindicates improvement in this client? 1 Staying clear of people who make him anxious Correct2 Talking with at least one person on the unit daily A client with recurrent episodes of depression comes to the mental health clinic for a routine follow-up visit. The nurse suspects that the client is at increased risk for suicide. What is a contributing factor to the client's risk for suicide? Psychomotor retardation 2 Decreased physical activity 3 Deliberate thoughtful behavior Correct4 Overwhelming feelings of guilt A client who is to begin a physical therapy regimen after orthopedic surgery expresses anxiety about starting this new therapy. Why does the nurse respond that some of this apprehension can be an asset? 1 Slow physiological function Increase alertness to the environment Many clients who call a crisis hotline are extremely anxious. The nurse answering the hotline phone remembers what characteristic as distinguishing posttraumatic stress disorders from other anxiety disorders? 1 Lack of interest in family and others Correct2 Reexperiencing the trauma in dreams and flashbacks A nurse knows that individuals who are alcoholics use alcohol for what reason? Correct1 Blunt reality A nurse is assessing a client with the diagnosis of schizophrenia, undifferentiated type. What defense mechanisms should the nurse anticipate that this client might use? Projection 2 Repression Correct3 Regression 4 Conversion When the nurse is managing the care of an acutely depressed client, which intervention demonstrates that the nurse recognizes the client's fundamental mental health need? Correct1 Role modeling a hopeful attitude regarding life and the future Anorexia nervosa follows a cyclical pattern. Place the following statements in order of progression through this cycle, with 1 as the first step and 4 as the last step. In what situation should a nurse anticipate that a client will experience a phobic reaction? 2 When thinking about the feared object Correct3 When coming into contact with the feared object During the intake interview at a mental health clinic, a client in withdrawal reveals to the nurse long-term, high-dose cocaine use. Which signs and symptoms support the conclusion that the client has been abusing cocaine for a prolonged time? Select all that apply. Correct1 Sadness Euphoria 3 Loss of appetite 4 Impaired judgment A client with a history of methamphetamine use is admitted to the mental health unit because of aggressive, violent behavior. For what clinical manifestations of methamphetamine use should the nurse assess this client? Select all that apply. 1 Correct3 Hyperthermia A 6-year-old child has been wetting the bed at night since the birth of a baby brother. What defense mechanism is the child using to cope with the stress of accepting a new family member? Correct1 Regression A nurse is teaching a client and family about the characteristics of dementia of the Alzheimer type. What physiologic characteristic should the nurse include? Periodic exacerbations 2 Aggressive acting-out behavior 3 Hypoxia of selected areas of brain tissue Correct4 Areas of brain destruction called senile plaques Clients with eating disorders often exhibit similar symptoms. What should the nurse expect an adolescent with anorexia nervosa to exhibit? Correct1 Affective instability A student is anxious about an upcoming examination but is able to study intently and does not become distracted by a roommate's talking and loud music. What level of anxiety is demonstrated by the student's ability to shut out the distractions? Correct1 Mild A nurse is caring for several clients with the diagnosis of bulimia nervosa. What primary feeling does the nurse anticipate that these clients experience after an episode of bingeing? Guilt A nurse is caring for several clients with the diagnosis of bulimia nervosa. What primary feeling does the nurse anticipate that these clients experience after an episode of bingeing? Guilt A woman with bipolar disorder, manic episode, has been spending thousands of dollars on clothing and makeup. She has been partying in bars every night and rarely sleeps or eats. The nurse in the outpatient clinic knows that this client rarely eats. What does the nurse recognize as the mostlikely cause of her eating problems? 1 Feelings of guilt Need to control others 3 Desire for punishment Correct4 Excessive physical activity A nurse is counseling the spouse of a client who has a history of alcohol abuse. What does the nurse explain is the main reason for drinking alcohol in people with a long history of alcohol abuse? Correct1 During a home visit to an older adult, the nurse observes a change in behavior and suspects delirium. The nurse assesses the client for one of several conditions that may have precipitated the delirium. Select all that apply. Correct1 Correct3 5 Restricted mobilit A nurse is caring for a client with the diagnosis of schizophrenia. What is a common problem for clients with this diagnosis? Chronic confusion Correct2 Disordered thinking During an assessment interview the client reports overwhelming, irresistible attacks of sleep. Which sleep disorder does the nurse conclude that the client is experiencing? 1 Insomnia Narcolepsy A nurse in an outpatient mental health setting has been assigned to care for a new client who has been found to have an antisocial personality disorder. What does the nurse expect to observe in the client during the assessment? 1 Correct3 Displays charm, has an above-average intelligence, and tends to manipulate others A nurse working on a substance abuse unit knows that the individual uses opioids most commonly for what reason? 1 Desires independence Correct2 Is trying to reduce stress A nurse interviews a young female client with anorexia nervosa to obtain information for the nursing history. What will the client's history most likely reveal? Correct1 Strong desire to improve her body image A client proclaims that he is "the second son of God." What type of delusion does the nurse identify? 1 Influence Correct2 Religious A nurse assesses a client recently admitted to an alcohol detoxification unit. What common clinical manifestation should the nurse expect during the initial stage of alcohol detoxification? Correct1 Nausea When visiting hours are over, a nurse approaches a client with paranoid schizophrenia, who shouts, "You're the one that made my lover leave me." What conclusion does the nurse make about the client? 2 The patient is actively hallucinating. Correct3 The patient feels a sense of vulnerability A client arrives at the mental health clinic disheveled, agitated, and demanding that the nurse "do something to make these feelings stop." What clinical manifestation is evident? Feelings of panic The nurse is caring for a client with dementia whose expression of emotions is altered. Which behavior is unexpected with this client? 1 Lability 2 Passivity Curiosity An obviously upset client comes to the mental health clinic and, after pushing ahead of the other clients, states, "I had an argument with my daughter, and now I'm tense, and worried, and angry." What level of anxiety does the nurse determine that the client is experiencing? 1 Mild 2 Panic Severe Correct4 Moderate A young adolescent is found to have anorexia nervosa. What does the nurse understand probably precipitated the anorexia nervosa? 1 2 Correct4 An inaccurate perception of hunger stimuli and a struggle between depende A mental health nurse is working on a unit where many clients have the diagnosis of alcoholism. Which defense mechanism does the nurse identify as most commonly used by clients who are alcoholics? Denial A client being admitted for alcoholism reports having had alcoholic blackouts. What is the best description of an alcoholic blackout? A fugue state resembling absence seizures 2 Fainting spells followed by loss of memory 3 Loss of consciousness lasting less than 10 minutes Correct4 Absence of memory in relation to drinking episodes The nurse is working with a client who has a diagnosis of borderline personality disorder. What personality traits should the nurse expect the client to exhibit? Select all that apply. Correct1 Engaging 2 5 Perfectionisti A nurse is assessing a client with a diagnosis of primary insomnia. Which findings from the client's history may be the cause of this disorder? Select all that apply. Significant life stress 2 Severe anxiety 3 Generalized pain Excessive caffeine 5 Chronic depression Environmental noise/distracto The nurse can identify the mostcommonly demonstrated comorbid disorders associated with generalized anxiety disorder (GAD) by assessing the client for which of the following? Select all that apply. 1 Obesity Correct3 Suicidal ideations 4 Impaired cognitive function Signs of alcohol withdrawa A recovering alcoholic joins Alcoholics Anonymous (AA) to help maintain sobriety. What type of group is AA? 1 Social group Correct2 Self-help group The nurse is interviewing the family about the onset of problems in a young client with the diagnosis of schizophrenia. In what stage of development does the nurse expect that the client's difficulties with reality testing began? 1 Puberty Correct2 Adolescenc An assistant to a dean at a local university is admitted to the psychiatric inpatient unit for assessment and treatment. The client claims to be president of the university. What type of delusion does the nurse identify? 1 Somatic Correct2 Grandiose A client with a recent history of binge eating and purging is admitted to the eating disorder unit with a diagnosis of bulimia nervosa. How does the nurse explain bulimia nervosa to the family? 1 3 Mood swings, ranging from euphoria to depression, associated with food Correct4 Uncontrollable ingestion of large quantities of food in a short period with subsequent purging A nurse is caring for a client with a diagnosis of catatonic schizophrenia. What clinical finding does the nurse expect the client to exhibit? 1 Crying 2 Self-mutilation Immobile posturing A nurse who plans to care for a client with an obsessive-compulsive disorder should understand that the client’s personality can usually be characterized in what way? Marked emotional maturity 2 Rapid, frequent mood swings 3 Elaborate delusional systems Correct4 Doubts, fears, and indecisiveness A nurse knows that individuals who are alcoholics use alcohol for what reason? Correct1 Blunt reality A nurse is counseling the spouse of a client who has a history of alcohol abuse. What does the nurse explain is the main reason for drinking alcohol in people with a long history of alcohol abuse? They are dependent on it. When caring for clients who are demonstrating manic behavior, the nurse must constantly reassess these clients' physical needs. What characteristic about these clients makes this particularly important? 1 Will withdraw to their rooms if left alone 2 Have difficulty making their needs known May gain too much weight from overeating Correct4 May become exhausted from excessive activity A client tells the nurse, "All my friends experiment with drugs. I like the high I get when I huff paint. Nothing bad is going to happen to me." What is the best response by the nurse? 1 "Inhalants can cause a mild state of intoxication." Correct2 "Huffing paint can damage your lungs, kidneys, and liver A client who is a regular user of cocaine is admitted to a rehabilitation facility. Which common side effects of regular cocaine use should the nurse expect when assessing this client? 1 Nausea, fatigue, and extreme hunger Correct2 A client is admitted to the hospital with a diagnosis of depression. What clinical manifestations of depression does the nurse expect when assessing this client? 1 2 What is the primary concern for a nurse caring for a client who is grossly impaired by stimulants? 1 Drowsiness Correct2 Seizure activity A nurse is interviewing a child with attention deficit disorder. For which major characteristic should the nurse assess this child? Correct1 Overreaction to stimuli A man has completed an alcohol detoxification program and is setting goals for rehabilitation. When the client sets outcomes, what need is it important for him to understand? 1 Plan to avoid people who drink. 2 Accept that he is a fragile person. Develop new social drinking skills. Correct4 Restructure his life without alcohol. A nurse is in the process of developing a therapeutic relationship with a client who has an addiction problem. What client communication permits the nurse to conclude that they are making progress in the working stage of the relationship? Select all that apply. 1 3 Correct5 Addresses how the addiction has contributed to family distress Many clients who call a crisis hotline are extremely anxious. The nurse answering the hotline phone remembers what characteristic as distinguishing posttraumatic stress disorders from other anxiety disorders? 1 Lack of interest in family and others Correct2 Reexperiencing the trauma in dreams and flashbacks A client with recurrent episodes of depression comes to the mental health clinic for a routine follow-up visit. The nurse suspects that the client is at increased risk for suicide. What is a contributing factor to the client's risk for suicide? Psychomotor retardation 2 Decreased physical activity 3 Deliberate thoughtful behavior Correct4 Overwhelming feelings of guilt A client with schizophrenia is admitted to an acute care psychiatric unit. Which clinical findings indicate positive signs and symptoms of schizophrenia? 1 Withdrawal, poverty of speech, inattentiveness 2 Flat affect, decreased spontaneity, asocial behavior Hypomania, labile mood swings, episodes of euphoria Correct4 Hyperactivity, auditory hallucinations, loose associations A nurse is caring for several clients with the diagnosis of bulimia nervosa. What primary feeling does the nurse anticipate that these clients experience after an episode of bingeing? Guilt What should a nurse conclude that a client is doing when he makes up stories to fill in blank spaces of memory? 1 Lying 2 Denying 3 Rationalizing Confabulating A client with a history of alcohol abuse says to the nurse, "Drinking is a way out of my depression." Which strategy will probably be most effective for the client at this time? Correct1 A self-help group A person with a history of alcoholism says, "I've been drinking since last Friday to celebrate my son's graduation from college." What defense mechanism does the nurse identify? 1 Denial 2 Projection Identification Correct4 Rationalization hat characteristic of the environment is most therapeutic for clients with the diagnosis of bulimia nervosa? 1 Controlling 2 Empathetic Focused on food Correct4 Based on realistic limits A nurse is interviewing a client newly admitted to an outpatient program after withdrawal from alcohol. What behavior best indicates that the client has accepted that drinking is a problem? 1 Participates in scheduled counseling sessions Attends Alcoholics Anonymous meetings daily A client who was admitted to the psychiatric unit because of a major depressive disorder is exhibiting increasingly withdrawn behavior. The nurse understands that eventually the client will experience what feelings? 1 A nurse in a mental health unit of the emergency department of a hospital frequently cares for adolescents who attempt suicide. What is important for the nurse to remember about adolescent suicide behavior? 1 3 Girls talk more about suicide before attempting it. Correct4 Boys are more likely to use lethal methods than are girls. A recovering alcoholic joins Alcoholics Anonymous (AA) to help maintain sobriety. What type of group is AA? 1 Social group Self-help grou A 56-year-old man is admitted to the inpatient unit after family members report that he seems to be experiencing auditory hallucinations. The man has a history of schizophrenia and has had several previous admissions. Which statement indicates to the nurse that the client is experiencing auditory hallucinations? 1 "Get these horrible snakes out of my room!" "I am not the devil! Stop calling me those names!" A client with a long history of alcohol abuse who has been hospitalized for 1 week tells the nurse, "I feel much better and probably won't need any more treatment." What does the nurse conclude when evaluating the client's progress? The client has accepted the illness and now must use willpower to resist alcohol. 2 Correct3 Anorexia nervosa follows a cyclical pattern. Place the following statements in order of progression through this cycle, with 1 as the first step and 4 as the last step. A client who is a polysubstance abuser is mandated to seek drug and alcohol counseling. What is an appropriate initial outcome criterion for this client? Correct1 Verbalizes that a substance abuse problem exists While a nurse is assisting with morning care for a client with the diagnosis of schizophrenia, the client suddenly throws off the covers and starts shouting, "My body is disintegrating! I'm being pinched." What term best describes the client's behavior? Correct1 Somatic delusion A 5-foot 5-inch (165 cm) 15-year-old girl who weighs 80 lb (36.3 kg) is admitted to a mental health facility with a diagnosis of anorexia nervosa. The nurse recognizes what factor as the most likely cause of her problem? A client with schizophrenia is admitted to an acute care psychiatric unit. Which clinical findings indicate positive signs and symptoms of schizophrenia? 1 3 Hypomania, labile mood swings, episodes of euphoria Correct4 Hyperactivity, auditory hallucinations, loose associations Nursing care for a client with a bipolar mood disorder, manic episode, is sometimes difficult. What important fact about these clients should the nurse consider when planning their care? 1 Embarrassed by their behavior Correct2 Aware of their environment and reality AWhat characteristic of an adolescent girl suggests to the nurse that she has bulimia? 1 History of gastritis 2 Positive self-concept Excessively stained teeth or success in a client’s alcohol treatment program? Correct2 Motivatio What is the greatest difficulty for nurses caring for the severely depressed client? Client's lack of energy 2 Negative cognitive processes 3 Client's psychomotor retardation Correct4 Contagious quality of depression It is determined that a staff nurse has a drug abuse problem. What approach to the staff nurse’s addiction should be taken as an initial intervention? 1 Counseled by the staf f psychiatrist 2 Dismissed from the job immediately Referred to the employee assistance program An adolescent with a conduct disorder is undergoing behavioral therapy in an attempt to limit behaviors that violate societal norms. What specific outcome criterion is unique to adolescents with this problem? 1 Increased impulse control 2 Identification of two positive personal attributes Demonstration of respect for the rights of others A severely depressed male client responds to therapy and with the help of the staff begins to set some daily objectives. Which behavior mostindicates improvement in this client? 1 Correct2 Talking with at least one person on the unit daily What is a priority nursing intervention in the care of a drug-dependent mother and infant? Correct1 Supporting the mother's positive responses toward her infant When planning care for a client who has just completed withdrawal from multiple- drug abuse, what reality in relation to the client should the nurse take into consideration? 1 Unable to give up drugs Unconcerned with reality Correct3 Unable to delay gratification What should a nurse identify as the most important factor in rehabilitation of a client addicted to alcohol? Correct1 Motivational readiness it with the diagnosis of anorexia nervosa. What typical signs and symptoms of anorexia nervosa does the nurse expect the client to exhibit? 1 Slow pulse, mild weight loss, and alopecia 2 Compulsive behaviors, excessive fears, and nausea An executive assistant experiences an overwhelming impulse to count and arrange the rubber bands and paper clips in his desk. The client feels that something dreadful will occur if the ritual is not carried out. Considering the client's symptoms, what does the nurse conclude about the rituals? 1 They are useful in our society as long as they can be controlled Correct2 They serve to control anxiety resulting from unconscious impulses A client who uses ritualistic behavior taps other clients on the shoulders three times as part of the ritual. What does the nurse infer about this client? 1 A blurred personal identity Poor control of sudden urges 3 A disturbance in spatial boundaries Correct4 A diminished ability to adapt to life's stresses On the third day of hospitalization, a client with a history of heavy drinking begins experiencing delirium alcohol withdrawal syndrome. What is the mostappropriate response by the nurse when the client begins experiencing hallucinations? 1 Withholding intervention, because the client may be having vivid dreams 2 Asking the client to describe the hallucinations and explaining that they are not real Administering the prescribed medication to the client to subdue the agitated be A nurse in the mental health unit is working with a group of adolescent girls with the diagnosis of anorexia nervosa. What does the nurse recall is the major health complication associated with intractable anorexia nervosa? 1 2 A client arrives at the mental health clinic complaining about feelings of extreme terror when attempting to ride in an elevator and feelings of uneasiness in large crowds. He reports that these fears are interfering with his concentration at work. What does the nurse identify as the source of these symptoms? 1 Conflict with society, resulting in an obsession 2 Depression about life events, resulting in unreasonable fears Generalized anxiety about conflicts, resulting in unreasonable fear nurse is caring for a client exhibiting compulsive behaviors. The nurse concludes that the compulsive behavior usually incorporates the use of which defense mechanism? Projection 2 Regression Correct3 Displacement 4 Rationalizatio What is the prognosis for a normal, productive life for a child with autism? 1 2 Correct4 Unlikely because of interference with so many parameters of function unit after attempting suicide. The client's history reveals that his first child died of sudden infant death syndrome 2 years ago, that he has been unable to work since the death of the child, and that he has attempted suicide before. When talking with the nurse he says, "I hear my son telling me to come over to the other side." What should the nurse conclude that the client is experiencing? 1 Fixed delusion 2 Magical thought Pathological regression Correct4 Command hallucinatio A nurse is making an assessment of a client's hallucinatory behavior. What is the most common type of hallucination? Visual 2 Tactile Correct3 Auditory nd grief counselors have been working with fellow students. What behaviors indicate to the school nurse that another student may be considering suicide? Select all that apply. Correct2 Giving away prized possessions What are the "four As" for which nurses should assess clients with suspected Alzheimer disease? Correct1 Amnesia, apraxia, agnosia, aphasia A newly admitted client is apathetic and exhibits an inappropriate affect. A diagnosis of schizophrenia is made. In light of the diagnosis, what is one symptom the nurse expects to identify in the client's communication or behavior? Logical deductions 2 Suicidal preoccupation 3 Absence of self-criticism Correct4 Response to internal stimulation A client with a history of gambling is experiencing legal difficulties for embezzling money and has been required to obtain counseling. During an intake interview the client says, "I never would have done this if I'd been paid what I am worth." What factor will create the greatest difficulty in helping this client develop insight? Feelings of boredom and emptiness 2 Grandiosity related to personal abilities Correct3 Projection of reasons for difficulties onto others An anxious client reports experiencing pain in the abdomen and feeling empty and hollow. A diagnostic workup reveals no physical causes of these clinical findings. What term best reflects what the client is experiencing? 1 Dissociation Correct2 Somatization A client is admitted to the mental health unit with the diagnosis of anorexia nervosa. What typical signs and symptoms of anorexia nervosa does the nurse expect the client to exhibit? 1 Slow pulse, mild weight loss, and alopecia 2 Compulsive behaviors, excessive fears, and nausea Amenorrhea, excessive weight loss, and abdominal distention What is a primary consideration for the nurse caring for a client with a history of substance abuse? Correct1 Setting firm, consistent limits and not varying from them client who has a long history of alcoholism has not worked for the past 10 years. When the nurse asks about daily activities the client responds, "I currently work in the office of a local construction company." Which mental mechanism should the nurse suspect that the client is using? 1 Regression Sublimation 3 Compensation Correct4 Confabulatio An older client with vascular dementia has difficulty following simple directions for selecting clothes to be worn for the day. What does the nurse identify as the cause of these problems? Receptive aphasia A client with a history of chronic alcoholism was admitted to a surgical unit after surgery to repair a severely fractured right ankle. The nurse is concerned that the client is experiencing manifestations of acute alcohol withdrawal when certain documentation and assessment data from the last 6 hours seem to indicate this problem. Which data are the cause of the nurse's concern? Select all that apply. Correct1 Tremors in both hands make it difficult for the client to hold a cup. A client experiencing nonspecific, excessive, unpleasant feelings of being worried concerning one's safety likely is experiencing which mental health disorder? Phobia 2 Panic disorder Correct3 Generalized anxiety disorder (GAD A mother brings her 5-year-old daughter to the children's clinic after teachers report that the girl is disobedient and hostile. The child has a negative attitude and argues often with her teachers. At this time she has not violated the rights of other students. The mother reports that she has also noticed this behavior at home. The nurse suspects that the behavior described is associated with what disorder? Anxiety disorder 2 Conduct disorder 3 Correct4 Oppositional defiant disorder On the third day of hospitalization, a client with a history of heavy drinking begins experiencing delirium alcohol withdrawal syndrome. What is the mostappropriate response by the nurse when the client begins experiencing hallucinations? 1 Withholding intervention, because the client may be having vivid dreams 2 Asking the client to describe the hallucinations and explaining that they are not real Administering the prescribed medication to the client to subdue the agitated behavior A 20-year-old homeless client at 38 weeks' gestation visits the prenatal clinic for the first time. She is accompanied by her 21-year-old boyfriend, who is the father of the baby. The nurse becomes concerned because as they sit in the waiting room, they are sneezing and yawning and have teary eyes. With what substance are these withdrawal signs associated? Heroin For what most common characteristic of autism should a nurse assess a child in whom the disorder is suspected? Responds to any stimulus 2 Responds to physical contact Correct3 Unresponsiveness to the environment nurse collaborates with a depressed client to increase self-esteem. What behavior should the nurse recall as typical of this type of client? One evening a nurse finds a client who has been experiencing persecutory delusions trying to get out the door. The client begs, "Please let me go. I trust you. The Mafia is going to kill me tonight." Which response is mosttherapeutic? A nurse is counseling a client who is experiencing substance abuse delirium. What communication strategies should be used by the nurse when working with this client? 2 Using humor when communicating with the client Correct3 Offering an introduction to the client at each me A client is experiencing hallucinations. What therapeutic intervention should the nurse plan to help the client cope with the hallucinations? 1 Reinforcing the perceptual distortions until the client develops new defenses 2 Providing an unstructured environment and assigning the client to a private room Correct4 What response from the nurse demonstrates an understanding of hallucinating behavior by a client? Correct1 Asking, "What are the voices telling you to do?" On the fifth day of hospitalization the nurse notes that a depressed client remains lying on the bed when the clients are called to the dining room for lunch. What should the nurse do to encourage the client to eat? Correct2 Offer to accompany the client to the dining room. he nurse is caring for a client with a somatoform disorder, conversion-type paralysis. What is the best nursing approach? Discussing topics other than the paralysis A nurse is assessing several depressed clients. Which behavior should alert the nurse to closely monitor a client for a suicide attempt? When the client does not eat Correct2 If the client describes a plan for suicid fter detoxification a client with a long history of alcohol abuse decides to attend Alcoholics Anonymous (AA) meetings at the hospital. On the day of the second meeting the client says, "I can’t go to the AA meeting today because I’m expecting an important phone call." What is the mosttherapeutic response by the nurse? Correct1 "You are expected to go to the meeting." A nursing home resident with dementia of the Alzheimer type, stage 2, who has been receiving donepezil is engaging in numerous acting-out behaviors. On what should the nurse base the initial plan of care? 1 Assessing the client’s level of consciousness Identifying the stressors that precipitate the client’s behavior A paranoid client is scheduled to begin group therapy. The client refuses to attend. What should the nurse do next? Correct1 Accept the client’s decision without discussion. An adolescent client with antisocial personality disorder plans to live with the parents after discharge. The parents request advice on how to respond to their child’s unruly behavior. What is the most therapeutic response by the nurse? "Discuss the behavior with your child and encourage the development of self-control." 2 "Avoid setting expectations for behavior and react to each situation as it arises." 3 Correct4 "Set clear limits, explain the consequences if your child disregards them, and firm In an outpatient mental health clinic a nurse is working with a client who is beginning to address more effective ways to handle stressful situations. The best nursing action to include in the plan of care is to have the client do what? Identify unhealthy habits that need to be altered. ewly diagnosed schizophrenia. What factor in the client’s history indicates a greater potential for recovery? Vague prepsychotic symptoms Ten minutes before lunch, a client with obsessive-compulsive behavior begins the ritual of changing clothes for the fourth time. How should the nurse respond to this behavior? Correct1 What should a nurse consider when planning care for a client who is using ritualistic behavior? 1 2 Correct4 The repeated thought or act A depressed client cries when the family does not visit. What is the mosttherapeutic response by the nurse? 1 "It’s difficult to realize that no one cares about you." Correct2 "Your family didn’t visit, and now you’re feeling rejected." A client comes to the mental health clinic for treatment of a phobia of large dogs. What should the nurse anticipate that this client will demonstrate? Correct1 Fear of discussing the phobia Resentmen A disturbed client says, "The voices are saying that I killed my husband." What is the best response by the nurse? "I just saw your husband, and he’s doing fine." 2 "Tell me more about your concerns for your husband." 3 "We’ll put you in a private room where you’ll be safe." Correct4 "You seem to be having very frightening thoughts right now." A client with the diagnosis of borderline personality disorder is manipulative and uses this behavior to get cigarettes from other clients. One day the client begins to badger another client. What should the nurse say while removing the client from the area? Correct1 "You must leave people alone; this behavior is unacceptable." A client with borderline personality disorder receives the wrong tray for lunch and scolds the dietary staff regarding this mistake. What is the most appropriate response by the nurse? "Getting angry is inappropriate; your behavior must stop." 2 "Yelling is unacceptable and will only get you placed in seclusion." 3 "You have to eat the first tray of food; then I’ll get another tray for you." Correct4 "It must be frustrating to get the wrong tray. I’ll order another tray for you." One day while shaving, a male client with the diagnosis of bipolar disorder tells the nurse, "I’ve hidden a razor blade, and tonight I’m going to kill myself." What is the best reply by the nurse? Correct1 "You’re going to kill yourself? A 17-year-old client is admitted to the hospital because of weight loss and malnutrition, and the primary healthcare provider diagnoses anorexia nervosa. After the client’s physical condition is stabilized, the provider, in conjunction with the client and parents, institutes a behavior-modification program. What component of behavior modification verbalized by parents leads the nurse to conclude that the parent has an understanding of the therapy? Correct1 Rewarding positive behavior An adult client with schizophrenia is involuntarily admitted to the psychiatric unit. While off the unit for needed testing, the client runs away. Legally, who should the nurse notify immediately? 2 3 Correct4 A nurse is planning activities for a withdrawn client who is hallucinating. What is the most therapeutic activity for this client? A client with schizophrenia, paranoid type, is readmitted to the hospital at the insistence of the family. While exploring feelings about the readmission, the client angrily shouts, "You’re one of them! Leave me alone!" How should the nurse respond? 2 3 Correct4 A 54-year-old has demonstrated increasing forgetfulness, irritability, and antisocial behavior. After the person is found disoriented and semi-naked while walking down a street, the diagnosis of dementia of the Alzheimer type is made. The client expresses fear and anxiety upon admission to a long-term care facility. What is the best nursing intervention in light of the client’s diagnosis? Correct2 Reassuring the client with the frequent presence of staf f members A nurse is assessing a client with bulimia nervosa. What should the nurse ask to obtain information about the client’s intake habits and patterns? 1 "Are you trying to control other people through the use of food?" "When you socialize, do you find that you eat more than when you eat by yourself?" 3 "Do you find yourself eating more right before the beginning of your menstrual cycle?" Correct4 "How frequently are you eating in A depressed older client has not been eating well since her admission to the hospital. The client repeatedly states, "No one cares." What is the mostappropriate response by the nurse? "We all care about you; now please eat." 2 "We all care about you; you have to eat to stay alive." Correct3 "I care about you. What are some foo A client comes to the hospital because of intense feelings of unrest, inability to sleep, and frequent episodes of panic. The client tells the nurse, "I admitted myself because I think I’m going crazy." What does the nurse identify the client’s remark to be? A plea for suppo An adolescent client with an antisocial personality disorder has been admitted to the hospital because of drug abuse and repeated sexual acting-out behavior. Which client behavior supports the nurse’s conclusion that actions directed toward modifying the behavior of this client have been successful? 2 3 Correct4 hat should the nurse keep in mind about rituals when planning care for a client who uses ritualistic behavior? Correct1 nurse plans to evaluate a newly admitted depressed client’s potential for suicide. What is the best approach to obtaining this information? Correct2 hyperactive 9-year-old child with a history of attention deficit–hyperactivity disorder is admitted for observation after a motor vehicle collision. On what should nursing actions be focused when the nurse is teaching about personal safety? 2 Correct3 A client with a history of obsessive-compulsive behaviors has a marked decrease in symptoms and expresses a wish to obtain a part-time job. On the day of a job interview the client arrives at the mental health center with signs of anxiety. What is the most therapeutic response to the client’s behavior by the nurse? 2 3 Correct4 college health clinic reporting increasing anxiety, loss of appetite, and an inability to concentrate. What is the most therapeutic response by the nurse? 2 "What have you identified as the cause of your anxiety?" Correct3 "It’s been difficult for you. How long has this been going on?" 4 "Let’s talk about your problems. Are you having difficulty client has been on the psychiatric unit for several days. The client arouses anxiety and frustration in the staff and manipulates them so well that staff members are afraid to approach the client. One morning the client shouts at the nurse, "You’ve worked it so I can’t go for a walk with the group today. You’re as cunning as a fox. I hate you! Get out, or I’ll hit you!" What is the best response by the nurse? 1 "Tell me what I did to upset you." "Go ahead and try to hit me if you need to." Correct3 A client in the mental health clinic has a phobia about closed spaces. Which desensitization method should the nurse expect to be used successfully with this client? Correct1 What should the nurse ask while assessing a Latina woman with depression for the risk of self-harm? Correct2 After a conference with the primary healthcare provider, a client with a borderline personality disorder cries bitterly, pounds the bed in frustration, and threatens suicide. What is the most helpful response by the nurse? 2 3 Correct4 The nurse notices that one of her clients, who has depression, is sitting by the window crying. What is the mostappropriate response by the nurse? 2 Correct3 A client whose depression is beginning to lift remains aloof from the other clients on the mental health unit. How can a nurse help the client participate in an activity? 1 Find solitary pursuits that the client can enjoy. 2 Speak to the client about the importance of entering into activities. Ask the primary healthcare provider to speak to the client about participating. Correct4 Invite another client to take part in a joint activity with the nurse and the client. An occupational health nurse is meeting with a new employee to obtain a health history and schedule an appointment with the nurse practitioner for a physical examination. How can the occupational nurse best respond when the new employee exhibits a moderate level of anxiety and verbalizes extreme nervousness about starting the new job? 1 "It’s common to feel a little nervous." "You’ll be less nervous when you get used to the job." 3 "I felt the same way when I first started working here." Correct4 "Feeling upset about starting a new job can be difficult. nurse notes that each time the primary healthcare provider or nurse manager visits, a disturbed client becomes extremely anxious. Today after a visit with the primary healthcare provider the client sits wringing the hands. What is the best initial response by the nurse? 1 "Tell me why you’re so stressed." Correct3 "I see that you’re wringing your han Continual pacing Correct2 Suspicious feelings 3 Inability to socialize with others The nurse is developing a plan of care for a client who is using ritualistic behavior. Initially the nurse must understand what about the ritual? That it is under conscious control 2 That it is used primarily for secondary gains 3 That it helps the client focus on the inability to cope with reality Correct4 That it helps the client control the level of anxiety the client is A nurse becomes aware of an older client’s feeling of loneliness when the client states, "I only have a few friends. My daughter lives in another state and couldn’t care less whether I live or die. She doesn’t even know I’m in the hospital." How should the nurse interpret the client’s communication? 1 As a call for help to prevent the client from acting on suicidal thoughts As a manipulative attempt to persuade the nurse to call the daughter Correct3 As a reflection of depression that is causing feelings of hopelessness Which intervention will the nurse implement when assisting a child with a history of aggressive behavior to regain control in the triggering phase of an assault cycle? 1 Correct2 Provide the child with a quiet, low-stimulus environmen A client with a prolonged history of chronic schizophrenia, paranoid type, shows the nurse a small plastic keychain and says that it provides protection from evil forces. The client then quickly hides the keychain, yelling, "Don’t take it away from me; it’s the only thing that protects me." How should the nurse respond? "You may keep it, because I know it’s important to you A nurse is admitting a client with a history of bipolar disorder. The nurse determines that the client is in the depressive phase of the disorder. Identify the signs and symptoms that support the nurse’s conclusion. Select all that apply. Correct1 Apathy Hyperactivity 3 Flight of ideas Loss of appetite Correct5 Sleep disturbanc What should the nurse do when an adolescent with the diagnosis of anorexia nervosa starts to discuss food and eating? 1 Listen to the client’s list of favorite foods and secure these foods for the client. A nurse is interviewing a client with a phobia. Which treatment should the nurse inform the client has the highestsuccess rate? 1 A client has recently undergone what was personally considered "a third unsuccessful cosmetic surgery." The primary healthcare provider diagnoses body dysmorphic disorder. What is the primary nursing objective? 2 Teaching the client about relaxation techniques Correct3 Exploring the issues that influence the client’s self-perception A client with a diagnosis of major depression refuses to participate in unit activities, claiming to be "just too tired." What is the best nursing approach? 1 Planning one rest period during each activity Explaining why the staf f believes that the activities are therapeutic 3 Encouraging the client to express negative feelings about the activities Correct4 Accepting the client’s feelings about activities calml client’s severe anxiety and panic are often considered "contagious." What action should be taken when a nurse’s personal feelings of anxiety are increasing? 1 2 A client with chronic depression has a history of suicidal ideations. Place the following nursing assessment questions in the appropriate order to best ensure client safety. What is most appropriate for a nurse to say when interviewing a newly admitted depressed client whose thoughts are focused on feelings of worthlessness and failure? most therapeutic for the nurse to say when assessing the adolescent’s suicide potential? he clients on a mental health unit go on a supervised day trip to a baseball game. When returning to the bus, a client with a narcissistic personality disorder insists on leaving the group to get an autograph from a player. What is the most appropriate response by the nurse? 2 Instructing the client in a loud voice to get on the bus so the group can go home Correct3 Informing the client in a matter-of-fact tone that everyone must remain with the group A client has been actively hallucinating for several days. What is the mosttherapeutic nursing intervention? 1 Asking the client who is speaking Correct2 Involving the client in simple activities on the unit A client exhibiting manic behavior is admitted to the psychiatric hospital. In which room should the nurse manager place the client? Correct1 One that has basic simple furnishing What should the nurse include when planning activities for an older nursing home resident with a diagnosis of dementia? 1 Varied activities that will keep the resident occupied Familiar activities that the resident can complete successfully An adult client confides to a clinic nurse, "I fantasize about having sex with children, and I get the urge to do it, too." What is the most appropriate response by the nurse? Correct1 Asking the client, "Have you ever acted on these thoughts?" A client comes to the mental health clinic with the complaint of a progressing inability to be in enclosed spaces. The primary healthcare provider makes the diagnosis of claustrophobia and prescribes desensitization therapy. The nurse recalls that desensitization therapy is used successfully with clients experiencing phobias because it is focused on what technique? Correct1 Imagery What is the priority nursing intervention in the planning of nursing care for an adolescent client with anorexia nervosa? Correct1 Rewarding weight gain by increasing privileges A client with cyclothymic disorder with hypomanic symptoms is admitted to the psychiatric unit. The client has progressively lost weight and does not take the time to eat the provided food. How can the nurse best respond to this situation? 1 By providing a tray in the client’s room By assuring the client that food is deserved Correct3 By ordering food that the client can hold and eat while moving around A nurse is caring for a client whose behavior is characterized by pathologic suspicion. What is the most therapeutic nursing action? 1 Providing distraction with reality-based activities Trying to establish trust through consistency of ca An older client is hospitalized with the diagnosis of dementia of the Alzheimer type. The son tearfully tells the nurse, "I should never have allowed my father to live alone like he wanted to do, but he hasn’t been this bad! I’m to blame! He didn’t even recognize me right off the bat." What response by the nurse is most therapeutic? 2 3 Correct4 A client is admitted to the acute care psychiatric unit with a diagnosis of panic disorder with agoraphobia. During the initial assessment phase, what should the nurse focus on? Correct1 Easing the client’s anxiety so further interviewing may be done diagnosed attention deficit–hyperactivity disorder (ADHD) in a pediatric client. When working with the family of this child, what should the nurse initially assess about the parents? History of the disorder 2 Relationship with each other Correct3 Attitudes about the diagnosis A nurse is caring for a newly admitted, extremely depressed client. What is the most appropriate initial goal for the client? 1 Setting realistic life goals Developing trust in others For the past week a young male client has been threatening other clients on the unit and trying to manipulate the female nursing staff members to obtain special privileges. What is the best nursing approach? 1 Assigning a nursing assistant to watch him when he is awake Ignoring the client’s behavior while protecting the vulnerable clients Correct3 Setting firm limits on the client’s behavior and consistently confronting him client on the psychiatric unit tells the nurse, "I’m a movie star, and the other clients are my audience." What is an appropriate conclusion for the nurse to document about what the client is experiencing? 1 Correct3 Delusion of grandeu hat is important when the nurse plans care for a client with paranoid ideation? 1 Avoiding placing demands on the client 2 Eliminating stress so that the client can relax Giving the client difficult tasks to provide stimulation Correct4 Providing the client with opportunities for nonthreatening social interaction he nurse working on the mental health unit finds a depressed client crying. What is the most therapeutic approach to help the client explore feelings? "Does crying help?" 2 "I know that you’re upset." Correct3 "Tell me what you’re feeling now. A client undergoing alcohol detoxification asks about attending Alcoholics Anonymous (AA) meetings after discharge. What is the nurse’s best initial reply? 1 "You’ll find that you’ll need their support." Correct2 "How do you feel about going to those meetings?" urse is interviewing a client with the diagnosis of dementia of the Alzheimer type. What question should the nurse ask to assess the client’s orientation to place? Correct1 "Where are you What client behavior indicates to the nurse that a client with schizophrenia, undifferentiated type, is improving and that the client’s plan of care can be updated? 1 Avoids other clients 2 Expresses negative feelings freely Describes delusions in meticulous detail Correct4 Communicates with others in an organized m client with bulimia nervosa eats two sandwiches, two salads, and four desserts for lunch. What client behavior should the nurse anticipate after the meal is consumed? Excessive exercise 2 Hoarding of more food for a later binge 3 Active socializing with small groups of clients Correct4 Withdrawing from the group to go to the bathroom client with the diagnosis of schizophrenia, paranoid type, is admitted to the hospital. The client says to the nurse, "I know they’re spying on me in here, too. I’m not safe anywhere!" What is the mosttherapeutic response by the nurse? 2 "Why do you feel they’d want to follow you here?" Correct3 "You don’t feel safe anywhere, not even in the hospital?" A clinically depressed client on a psychiatric unit of a local hospital uses embroidery scissors to cut the wrists. After treatment, when the nurse approaches, the client is tearful and silent. What is the best initial intervention by the nurse? 1 Note the client’s behavior, record it, and notify the primary healthcare provider. Sit quietly next to the client and wait until the client begins to speak. 3 Correct4 Comment, "I notice that you seem sad. Tell me what it’s like A client is heard saying, "I like eggs, fried by Meg, served on a keg, kicked in the leg and don’t want her to hoopanize them ever again." What should the nurse note in the client’s record? 1 "Word salad and echolalia appear to be the preferred speech patterns." Correct2 "Client demonstrates clang association and neologism speech patterns." A 22-year-old client with antisocial personality disorder is being discharged and is to continue psychotherapy on an outpatient basis. When evaluating the client’s chance of improvement, what should the nurse anticipate? 1 That the client’s prognosis for adjusting to a limited lifestyle is excellent 3 Correct4 That the client’s ability to change will be limited unless there is a re A client is admitted to the hospital with the diagnosis of severe anxiety. What should the nurse’s plan of care for a client with an anxiety disorder include? 1 Promoting the suppression of anger by the client Correct2 Supporting the verbalization of feelings by the client What should the nurse do when planning continuing care for a moderately depressed client? 1 Encourage the client to determine leisure time activities. Correct2 Offer the client the opportunity to make some decisions. urse notes that a client with dementia tries to cope with anxiety by using confabulation. What does the nurse plan to teach the family about confabulating? 1 The client may fantasize about past experiences. The client has poor control of disorganized thoughts. Correct3 The client will make up what cannot be remembered. A client on a psychiatric unit who has been acting out for several weeks approaches the nurse and says, "I’m really sorry about how I’ve acted. I’ll bet everyone thinks I’m an idiot." What is the best initial response by the nurse? "You’re wondering how others will react to you now." young adult client is admitted to the hospital with a diagnosis of schizophrenia, paranoid type. The client has been saying, "The voices in heaven are telling me to come home to God." What should initial nursing care be focused on? 1 Correct2 Potential for self-harm A depressed client tells the nurse, "I don’t get out of bed most mornings because I don’t feel like it." What is the best reply by the nurse? 1 "Try going to bed earlier so you feel more rested." "Most people don’t like getting out of bed in the morning." 3 "Try to spend 1 hour in bed, relaxing and preparing for the day." Correct4 "Getting up and involved in an activity should help lift your mood." A 20-year-old college student reports to the nurse at the college health clinic increasing anxious feelings, inability to sleep, and a loss of appetite. The client also reports an inability to concentrate and a drop in grades. What question should the nurse ask? 1 "With whom have you shared your feelings of anxiety?" 2 "What have you identified as the cause of your anxiety?" "It must be difficult for you. How long has this been going on?" In her eighth month of pregnancy, a 24-year-old client is brought to the hospital by the police, who were called when she barricaded herself in a ladies’ restroom of a restaurant. During admission the client shouts, "Don’t come near me! My stomach is filled with bombs, and I’ll blow up this place if anyone comes near me." What does the nurse conclude that the client is exhibiting? 1 2 A 3-year-old child has a tentative diagnosis of pervasive developmental autistic disorder. Which characteristic would the nurse find most unusual when assessing this toddler? 1 Interest in music 2 Ritualistic behavior Attachment to odd objects Correct4 Responsiveness to the parents Which activity is most appropriate for a nurse to introduce to a depressed client during the early part of hospitalization? 1 Board game Correct2 Project involving drawing When talking with one of the day nurses, a client with the diagnosis of anorexia nervosa states that the day nurses give better care and are nicer than the night nurses. The client also asks a question that the day nurse knows was already answered by one of the night nurses. What conclusion should the nurse make about the client? 1 The client needs assistance in exploring and verbalizing feelings about the night nurses. The client is trying to develop a bond of trust with a staf f member that should be supported. Correct3 Nurses on a psychiatric unit have secluded a client who has the diagnosis of bipolar I disorder, manic episode, and who has been losing control and throwing objects while in the dayroom. What is the mostimportant intervention for the client who is given an as-needed (PRN) medication and confined to involuntary seclusion? 1 Continue intensive nursing interactions. Correct2 Evaluate the client’s progress toward self-control. A nurse is accompanying a client with a diagnosis of anxiety disorder who is pacing the halls and crying. When the client’s pacing and crying worsen, the nurse suddenly feels uncomfortable and experiences a strong desire to leave. What is the most likely reason for what the nurse is experiencing? Correct1 An empathic communication of anxiety A client on a psychiatric unit misses breakfast because of an elaborate handwashing ritual. What is the mostimportant therapeutic intervention during the early period of the client’s hospitalization? 1 Having the client wait until after breakfast to start the ritual Correct2 Waking the client early so the ritual can be completed before breakfast The ritual of a client with obsessive-compulsive disorder involves washing the hands every 30 minutes. The client becomes anxious and agitated if unable to perform this ritual. What should the nurse in the mental health daycare center do? 1 Lock the door to the bathroom. Keep the client actively involved in projects in the facility. 3 Correct4 Set a contract with the client limiting the frequency of the ritual. A client with a mood disorder is being discharged from a psychiatric hospital after agreeing to continue follow-up visits with a therapist. During the last interview with the nurse before discharge, the client says, "I’ve told you a lot about my life and my problems, but there are a few things that bother me that I’ve told no one." What is the most therapeutic response by the nurse once it has been determined that the client is not at risk for self-harm or harming others? 1 "The purpose of our getting together is to discuss your problems." "Do you want to work on those during the few minutes we have left?" 3 "What kind of problem have you not shared with me during our time together?" Correct4 "One purpose of continuing counseling is to allow you to discuss things that bother you." A nurse is caring for a client with the diagnosis of alcohol withdrawal delirium. Which action is mostappropriate for the nurse to implement? 1 Keeping the client calm by applying wrist restraints Encouraging the client to relate the content of hallucinations Correct3 Assuring the client that the symptoms are part of the withdrawal syndrome A nurse who works in a mental health facility determines that what is the priority nursing intervention for a newly admitted client with bulimia nervosa? Correct1 Check on the client continually. A forgetful, disoriented client is exhibiting inappropriate behaviors signifying dementia. Nursing management should be directed toward A forgetful, disoriented client is exhibiting inappropriate behaviors signifying dementia. Nursing management should be directed toward what goal? 1 2 Correct4 Rechanneling the client’s energies into more appropriate behaviors What client response should the nurse anticipate when an attempt is made to prevent a client from carrying out ritualistic behavior? 1 Relief Correct2 Ange client with bipolar mood disorder, manic episode, says to the nurse, "I don’t know what I’m doing here. I never felt better in my life; I’ve got the world on a string around my finger." What is the nurse’s most therapeutic response to this comment? 1 "Have you ever felt this way before?" Correct2 "You’re feeling pretty elated right now." A middle-age client who has lost 20 lb (9.1 kg) over the last 2 months cries easily, sleeps poorly, and refuses to participate in any family or social activities that were previously enjoyed. What is the most important nursing intervention? 1 Providing the client with a high-calorie, high-protein diet 2 Reducing the client’s crying episodes by setting firm, consistent limits Correct4 Allowing the client to externalize feelings, especially anger, in a safe manner adolescent who is extremely underweight and disappears into the bathroom after meals angrily says to the nurse, "I don’t need to be here. I don’t have any problems. Stop watching me." What is the mosttherapeutic response by the nurse, aimed at reducing the client’s feeling of being threatened? Correct1 "I hear how frustrated you are to be here." When working with a client who has a phobia of black cats, what problem does the nurse anticipates for this client? 1 Denying that the phobia exists Anger toward the feared object Correct3 Anxiety when discussing the phobia A client with the diagnosis of panic disorder jumps when spoken to, complains of feeling uneasy, and says, "It’s as though something bad is going to happen." Which action by the nurse is most therapeutic at this time? Correct1 Stay with the client to be a calming presence. Without knocking, a nurse enters the room of a young male client with the diagnosis of panic disorder and finds him masturbating. What should the nurse do? Correct1 Apologize and leave the room. with symptoms of manic behavior has pressured speech punctuated with profanity. What is the most therapeutic approach for the nurse to use to manage this client’s behavior? Correct2 Stating that the use of profanity should stop, because it is inappropriate During an assessment the nurse realizes that the client is experiencing a hallucination when the client says what? 1 "I am going to save the world because I am God." Correct2 "My insides smell like they’re going to just rot away." After 2 days on the unit a client with the diagnosis of schizophrenia refuses to take a shower. What is the mostappropriate intervention by the nurse? 1 Having the staf f give the client a shower 2 Simply stating that the client must shower now Gently pointing out that the client’s appearance is upsetting the other clients Correct4 Gently asking whether the client would wash the hands and face if given a basin of water A cachectic adolescent with the diagnoses of anorexia nervosa, dehydration, and electrolyte imbalances is admitted to a mental health facility. The adolescent has been obsessed with weight, has exercised for hours every day, has taken enemas and laxatives several times a week, and has engaged in self-induced vomiting. What outcome is a priority for the nurse planning care for this client? 1 Identifying personal strengths 2 Controlling impulsive behaviors Correcting electrolyte imbalances A depressed client tells a nurse, "I want to die." What is the most therapeutic response by the nurse? Correct1 "You would rather not live." A male client with the diagnosis of antisocial personality disorder takes a female nurse by the shoulders, kisses her, and shouts, "I like you." What is the most appropriate response by the nurse? "Thank you. I like you, too." 2 "I wish you wouldn’t do that." 3 "Don’t ever touch me like that again. I don’t like it " Correct4 "Your behavior is inappropriate. Don’t do that again. A client with a diagnosis of obsessive-compulsive disorder is frequently late for appointments because it takes so much time each day to complete a ritualistic handwashing routine. What is the most therapeutic nursing intervention? Correct1 Accepting the client’s ritual in a matter-of-fact manner without criticism A nurse recalls that the environment is important when caring for a client with the diagnosis of bipolar II disorder with hypomanic episodes. What should the nurse do when caring for clients with this disorder? Correct1 Provide a quiet atmosphere by placing the client in a private room. Schizophrenia is associated with both positive and negative symptoms. While assessing a client with schizophrenia, the nurse notes that the client is experiencing positive symptoms; what does the nurse observe that leads to this conclusion? Select all that apply. 1 Poverty of speech Correct4 Correct5 Auditory hallucinations ow should the nursing staff fulfill the nutritional needs of a client experiencing periods of extreme mania and hyperactivity? 1 Accepting that the client will eat when hungry 2 Allowing the client to prepare meals to eat when desired Frequently offering high-calorie snacks that the client c An adult client charged with molesting a child is admitted for psychiatric evaluation. When a nurse invites the client to come to dinner, the client refuses and says, "I don’t want anyone to see me. Leave me alone." What is the best response by the nurse? 1 "Certainly. I’ll respect your wishes." "You sound upset; let’s talk about it. A client with bipolar disorder is exhibiting accelerating activity and flight of ideas. What is the best nursing intervention to limit the accelerating manic behavior? 1 Involving the client in a video game 2 Correct4 Engaging the client in conversation while walking slowly in the hall client with bipolar I disorder, manic episode, is admitted to the mental health unit of a community hospital. When developing an initial plan of care for this client, what should the nurse plan to do? Increase the client’s gym time. 2 Isolate the client from peers. Correct3 Encourage increased nutritional intak A nurse is assigned to care for a client with the diagnosis of schizophrenia who is hallucinating. What is the first consideration in trying to establish a trusting relationship? 1 Family members must be included in the plan of care. 2 The client cannot be distracted from the hallucinations. The client adamantly believes what is being experienced. At times a client’s anxiety level is so high that it blocks attempts at communication and the nurse is unsure of what is being said. To clarify understanding, the nurse says, "Let’s see whether we mean the same thing." What communication technique is being used by the nurse? 1 Reflecting feelings 2 Making observations A depressed, withdrawn client exhibits sadness through nonverbal behavior. What should the nurse plan to help the client to do? 1 Increase structured physical activity. Correct2 Cope with painful feelings by sharing them. The nurse is caring for a client who is using paranoid ideation. When planning care, what should the nurse remember the importance of? 1 Not placing demands on the client 2 Removing stress so the client can relax Giving the client difficult tasks to provide stimulation Correct4 Providing the client with activities that are structured an A client is admitted to the psychiatric service with a diagnosis of severe depression. When approached by the nurse, the client says, "You know I’m a sorry, lazy person. I don’t deserve a job. I’m just stupid and no good." What does the nurse conclude that the client is experiencing? 1 Nihilistic delusions Delusions of persecution Correct3 Feelings of self-deprecation A depressed client arrives at the mental health unit with mild suicidal ideation but no plan of action. Assessment data reveal that the client has many family responsibilities and adequate family support and attends church regularly. What does the nurse determine about this client? 2 3 Correct4 A nurse speaks with a client who has just experienced a panic attack. Which statement will be the most therapeutic in addressing the client’s concerns? 1 Correct2 client who has severe anxiety starts to cry while talking with the nurse. The client is so upset that the crying becomes uncontrollable. What is the best response by the nurse? 1 Correct2 A client with schizophrenia is admitted to a psychiatric unit. The client is talking while walking in the hall, is unkempt, and obviously has not washed in several days. What should the nurse say when trying to help this client shower? Correct2 A depressed client is admitted to the mental health unit. What factor should the nurse consider most important when evaluating the client’s current risk for suicide? 2 Lack of interest in appearance 3 How long the depression has existed Correct4 Impending anniversary of the loss of a loved one client has the diagnosis of histrionic personality disorder. Which behavior should the nurse expect when assessing this client? 1 Boastful and egotistical Rigid and perfectionistic Correct3 Extroverted and dramatic What therapy has the highest success rate for people with phobias? Correct1 Desensitization involving relaxation techniques A client who uses a ritual of counting paper in the printer tells the nurse, "I’m spending 30 minutes counting each time I make copies, and my boss is getting very upset. What should I do?" What is the best response by the nurse? Correct1 "Limit photocopying by clustering it to two or three times a day A nurse begins a therapeutic relationship with a client with the diagnosis of schizotypal personality disorder. What is the best initial nursing action? 1 Setting limits on manipulative behavior Correct3 Respecting the client’s need for social isolatio A psychiatric nurse has been working with a client who is experiencing a relapse of psychotic symptoms. Command hallucinations are ruled out, and the content of the auditory messages has been determined. What should the nurse’s next planned intervention be? 1 Teaching the client how to prevent relapses Instructing the client to eliminate dietary stimulants 3 Helping the client learn strategies for disregarding the voices Correct4 Assisting the client in recognizing hallucinations when they occur A nurse is orienting a new client to the unit when another client rushes down the hallway and asks the nurse to sit down to talk. The client requesting the nurse’s attention is manipulative and uses acting-out behaviors when demands go unmet. How should the nurse intervene? 1 By suggesting that the client requesting attention speak with another staf f member By leaving the new client, saying, "I’ll talk with the other client until things calm down." 3 Correct4 By saying to th A client is admitted to the mental health unit with the diagnosis of major depressive disorder. Which statement alerts the nurse to the possibility of a suicide attempt? 1 During a group discussion it is learned that a group member hid suicidal urges and committed suicide several days ago. What should the nurse leading the group be prepared to manage? 1 Guilt of the co-leaders for failing to anticipate and prevent the suicide 2 Guilt of group members because they could not prevent another’s suicide Lack of concern over the suicide expressed by several of the members in the group Correct4 A depressed client often sleeps past the expected time of awakening and spends excessive time resting and sleeping. Which nursing intervention is appropriate for this client? Correct1 Restricting the client’s access to the bedroom What is the best nursing intervention during the working phase of the therapeutic relationship with which to meet the needs of individuals who demonstrate obsessive-compulsive behavior? Restricting their movements 2 Calling attention to the behavior 3 Keeping them busy to distract them Correct4 A client has been attending weekly outpatient psychotherapy sessions for several months. The nurse psychotherapist has been working with the client to help lessen obsessive-compulsive behaviors that have interfered with the client’s work performance. What information about the client best validates the client’s improvement? 1 2 Correct4 Receives a letter from a supervisor at work stating job performance has improved A nurse sits with a depressed client twice a day, but there is little verbal communication. One afternoon the client asks, "Do you think they’ll ever let me out of here?" What is the best reply by the nurse? 1 "We should ask your primary healthcare provider." "Everyone says you’re doing fine." 3 "Do you think you’re ready to leave?" Correct4 "How do you feel about leaving here?" A nurse at the mental health center has been counseling the family of an adolescent client with anorexia about nutrition. Which statement made by a family member demonstrates an adequate understanding of the needs of the client? 1 "We won’t have to worry about this passing fad for long." "We’ll monitor both of our teenagers’ exercise habits closely." 3 Correct4 "We should give our child more input into our meal planning." n an effort to foster a healthy grief response to the birth of a stillborn child, how should the nurse respond to the mother’s questions about the cause? 1 "This often happens when something is wrong with the baby." "It’s God’s will; we have to have faith that it was for the best." 3 "You’re young, and you’ll have other children—wait and see." Correct4 "You may be wondering whether something you did caused this." Risk for assaultive behavior is highestin the mental health client who does what? 1 Uses profane language 2 Touches people excessively Exhibits a sudden withdrawal Correct4 Experiences command hallucinations A nurse is obtaining a health history from a client who is known to be verbally abusive. The client tells the nurse, "You’re ugly, and you’re probably stupid, too. Why am I stuck with you as my nurse?" What is the best response by the nurse? 1 "It doesn’t matter what you think, because I know I’m a capable nurse." 2 "Tell me more about why my caring for you today is so upsetting to you." Correct4 "You are talking inappropriately, so I’m going to leave and client is admitted to the acute psychiatric unit of the local community hospital. The client is guarded and suspicious. After a thorough evaluation, a diagnosis of schizophrenia, paranoid type, is made. What initial approach should be used by the nurse assigned to establish a therapeutic one-on-one relationship with this client? Correct1 Casual and honest 2 Warm and friend nurse is caring for a client who is confused and delirious. What is the most therapeutic intervention when the nurse is interacting with this client? 1 Reassuring the client that the client will get better Correct2 Directing the client’s daily activities on the unit A hyperactive, acting-out 9-year-old boy is started on a behavior modification program in which tokens are given for acceptable behavior. When he begins to lose a game he is playing with other children, he begins to kick the other children under the table and call them names. What is the most appropriate behavior modification technique for the nurse to use? Ignoring the child’s behavior Correct2 Placing the child in a time-out A nurse is using cognitive therapy to help a client who experiences panic attacks. What is the goal of this therapy? 1 Preventing future panic attacks 3 Stopping the panic attacks once they begin Correct4 Decreasing the fear of having panic attacks hen a nurse enters a room to administer an oral medication to an agitated and angry client with schizophrenia, paranoid type, the client shouts, "Get out of here!" What is the most therapeutic response? 1 Stating, "You must take your medicine now." Saying, "I’ll be back in a few minutes so we can talk." 3 Explaining wh A male adolescent with the diagnosis of antisocial personality disorder spends a great deal of time with a female adolescent client on the unit. One day the nursing assistant enters the female client’s room and finds them in bed together. The nursing assistant reports the incident to the nurse. What should the nurse do? 1 Lock the bedroom doors. 2 Assign the same staf f member to observe both clients several times an hour. Call a unit meeting to talk about sexual activity among the clients on the unit. Correct4 t with the diagnosis of paranoid schizophrenia refuses to eat at mealtime. What nursing action is mostbeneficial? 1 Correct2 Giving the client food in unopened packages When a nurse sits next to a depressed client and begins to talk, the client responds, "I’m stupid and useless. Talk with the other people who are more important." Which response is mosttherapeutic? 1 "Everyone is important." "Do you feel that you’re not important?" 3 "Why do you feel that you’re not important?" Correct4 "I want to talk with you because you are important to me." client with acute schizophrenia tells the nurse, "Everyone hates me." What is the best response by the nurse? "Tell me more about this." What is the priority nursing intervention in the planning of nursing care for an adolescent client with anorexia nervosa? Correct1 Rewarding weight gain by increasing privileges nurse is assigned to care for a college student who has been talking to unseen people and refusing to get out of bed, go to class, or participate in daily grooming activities. What is the nurse’s initial intervention for this client? 1 Providing frequent rest periods 2 Reducing environmental stimuli Facilitating the client’s social relationships with a peer group Correct4 Attempting to establish a meaningful relation An adolescent with the diagnosis of anorexia nervosa is admitted to the psychiatric unit of a local hospital. What should the nurse include in the plan of care? 1 Limited opportunities for decision-making Correct2 Provision of supervision during and after mealtime A client tells a nurse, "I have been having trouble sleeping and feel wide awake as soon as I get into bed." Which strategies should the nurse teach the client that will promote sleep? Select all that apply. 1 Eating a heavy snack near bedtime 2 Reading in bed before shutting out the light Leaving the bedroom when unable to sleep 4 Drinking a cup of warm coffee with milk at bedtime Exercising in the afternoon rather than in the evening Correct6 Counting backward from 100 to 0 when the mind is racing When caring for a newly admitted depressed client, a nurse arranges for a staff member to remain with the client continuously. What information supports the nurse’s decision to institute this precaution? Select all that apply. 1 Refusal to eat any food Correct3 Correct5 Statements that life is not worth living A nurse is caring for a client with the diagnosis of schizophrenia, paranoid type. What should the nurse plan for the client’s initial care? 1 Discussing important life events Providing a nonthreatening environment While a nurse is talking with a client, another client comes up and shouts, "I hate you! You’re talking about me again!" and throws a glass of juice at the nurse. What is the best response by the nurse to this outburst? 1 Repeating the client’s words and asking for clarification Removing the client from the room because limits must be placed on such behavior he nurse is acting as group leader for the weekly gathering of clients with bipolar disorder and their families. When the wife of one client expresses concern that, "he’s not taking the medications right and will never get better," other family members begin to express their concerns about medication effectiveness. Several clients respond that the family members just don’t understand what they are dealing with. Place the following nursing interventions in the appropriate order to best address the issues being expressed. On entering a depressed client’s room one morning, the nurse finds the client still in bed. The client says, "I can’t get dressed and go to breakfast." How should the nurse respond? 1 "You can’t just lie in bed. You need to get up now and go to breakfast." 2 "I’ll get you dressed. I understand that you have difficulty helping yourself." "Promise me you’ll get dressed for lunch. If you do I’ll let you stay here in bed." Correct4 "Take your time. It is not necessary to hurry, and I’ll help you if you need me to." A client with a history of schizophrenia attends the mental health clinic for a regularly scheduled group therapy session. The client arrives agitated and exhibits behaviors that indicate that the client is hearing voices. When a nurse begins to walk toward the client, the client pulls out a large knife. What is the best approach by the nurse? Correct1 Firm The nurse determines that the plan for bolstering an overweight adolescent’s self- esteem has been effective when, 3 months later, the adolescent’s mother reports that the adolescent is doing what? 1 Seems to be doing average work in school Has asked her how to bake bread and cookies 3 Correct4 Joined a dirt bike group that meets at the school A frail, depressed client who frequently paces the halls becomes physically tired from the activity. What action should the nurse take to help reduce this activity? 1 Restrain the client in a chair. Correct2 Have the client perform simple, repetitive tasks. woman with bipolar disorder, manic episode, has been spending thousands of dollars on clothing and makeup. She has been partying in bars every night and rarely sleeps or eats. The nurse in the outpatient clinic, knowing that this client rarely eats, recognizes that her eating problems most likely result from what? 1 Feelings of guilt 2 Need to control others Desire for punishment Correct4 Excessive physical activity A nurse’s best approach when caring for a confused older client is to provide an environment with what? Space for privacy 2 Group involvement Correct3 Trusting relationships 4 A depressed client is brought to the emergency department after taking an overdose of a sedative. After lavage the client says, "Let me die. I’m no good." What is the most appropriate response by the nurse? 1 Correct2 "You must have been upset to try to take your life." An extremely agitated client hospitalized in a mental health unit begins to pace around the dayroom. What should the nurse do? 1 Lock the client in the client’s room to limit external stimuli. Let the client pace in the hall away from other clients. During the first month in a nursing home, an older client with dementia demonstrates numerous disruptive behaviors related to disorientation and cognitive impairment. What should the nurse take into consideration when planning care? 1 Client’s orientation to time, place, and person Ability to perform daily activities without assistance from others Correct3 Stressors that appear to precipitate the client’s disruptive behavior A deeply depressed, withdrawn client remains curled up in bed and refuses to talk to the nurse. What should the nurse do initially to break through the client’s withdrawal? Correct1 Sit with the client for set periods each hour. When a disturbed client who has a history of using neologisms says to the nurse, "My lacket huss kelong mon," how should the nurse respond? 1 Correct2 TellinSuicide precautions are prescribed for a newly admitted client. What is the most therapeutic way to provide these precautions? 1 Correct3 A nurse plans to establish a trusting relationship with a client who is using paranoid ideation. How should the nurse begin to accomplish this? Correct1 By being available on the unit but waiting for the client to approach A client is admitted to the mental health clinic for treatment of an obsessive- compulsive disorder that impairs ability to work outside the home. What should the nurse consider about the client’s behavior when developing a plan of care? Correct1 The client knows that the behavior is illogical but is unable to stop it A nurse who is working on a psychiatric unit notes that a client with schizophrenia is beginning to pace around the lounge while glaring at other clients. How should the nurse respond to this behavior? 1 By pointing out the behavior to the client Correct2 By walking with the client to a quiet area on the unit A 5-year-old with attention deficit–hyperactivity disorder (ADHD) exhibits a short attention span and demonstrates intermittent head-banging and hair-pulling, as well as excessive motor activity. What is the priority nursing objective for this child? Correct2 Maintaining safe A 22-year-old client with the diagnosis of schizophrenia has been in a mental health facility for approximately 2 weeks. After the parents visit the client is seen pacing in the hall, talking loudly alone. What should the nurse’s initial intervention be? 1 Obtaining a prescription for a tranquilizer Asking the client about the events of the day Which nursing intervention is indicated for a client with an anxiety disorder? 1 Encouraging suppression of anger by the client Correct2 Promoting verbalization of feelings by the client nurse is caring for a client who is experiencing major depression. What feeling should the nurse anticipate that the client will likely have difficulty expressing? 1 Need for comforting Correct2 Anger toward other An older adult resident of a nursing home who has the diagnosis of dementia of the Alzheimer type frequently talks about the good old days at the ranch. What is the most appropriate action by the nurse? Correct1 Allowing the resident to reminisce about the past and listening with interest A client whose spouse recently died appears extremely depressed. The client says, "What’s the use in talking? I’d rather be dead. I can’t go on without my spouse." What is the best response by the nurse? 1 Correct3 "Are you thinking about killing yourself?" A client with the diagnosis of bipolar disorder, manic episode, attends a mental health day treatment program. What supervised activity will be most therapeutic for this client during the early phase of treatment? Doing a needlepoint project 2 Joining a brief swimming competition Correct3 Walking around the facility with a nurse A daycare environment is recommended for a client with incapacitating behaviors resulting from an obsessive-compulsive personality disorder. The client’s partner asks the nurse why this approach is necessary. What is the best response by the nurse? 1 "This environment limits time to carry out the rituals." Correct2 "A neutral atmosphere facilitates the working through of conflicts A client in a mental health facility with the diagnosis of bipolar disorder, manic phase, is argumentative, domineering, and exhibitionistic. A visitor reports that this client is running down the hall, scaring people. What should the nurse do first? 1 Ask the client the reason for running down the hall. Assess the client’s behavior in a nonthreatening manner A client comes to a trauma center reporting that she has been raped. She is disheveled, pale, and staring blankly. The nurse asks the client to describe what happened. What is the nurse’s rationale for doing this? 1 2 The parent of a child with a tentative diagnosis of attention deficit–hyperactivity disorder (ADHD) arrives at the pediatric clinic insisting on getting a prescription for medication that will control the child’s behavior. What is best response by the nurse? Correct1 "It must be frustrating to deal with your child’s behavior." A client in an acute mental health unit appears severely depressed. The client does not initiate conversations or perform personal care. Questions are answered with a barely audible one- or two-word response. The nurse sits with the client and makes no demands. On what premise is the nurse’s intervention for this client based? 1 Nurses are required to spend time with assigned clients. 2 Environmental stimulation helps depressed clients feel more worthwhile. Nurses are expected to initiate one-on-one interactions on an acute care unit. Correct4 Spending time with depressed clients demonstrates that they are worthy of attention. After a week on the mental health unit, a client with the diagnosis of paranoid schizophrenia continues to say, "They’re trying to kill me. They all are." What is the best response by the nurse? 1 2 Correct3 "You’re having very frightening thoughts." When planning nursing care for a client with severe agoraphobia, what should the nurse do first? Correct1 Determine the client’s degree of impairment. essential intervention should be included in the plan? 1 Assessing the symptoms and teaching the client about the disorder 2 Encouraging participation in cognitive and social skills enhancement Maintaining a daily routine and instituting family and group therapies Correct4 Instituting psychopharmacologic prescriptions and supportive communication A 2-year-old boy’s mother attempts suicide and is admitted to a mental health facility. What is the nurse’s priority when planning care for this client? 1 Supporting parental skills Ensuring the child’s safety Correct3 Maintaining constant observation of the client What is the nurse’s primary outcome goal when managing the care of a client diagnosed with generalized anxiety disorder (GAD)? 1 Creating an anxiety-free environment for the client One evening an older client with a diagnosis of dementia chokes on a piece of food and becomes panicky and cyanotic. The nurse performs the abdominal thrust maneuver, and a bolus of food pops out of the client’s mouth. After several deep respirations, the client’s cyanosis passes. What is mostappropriate for the nurse to do next? 1 Correct3 Touch the client’s hand while providing verbal support. While the nurse is talking to a hypermanic client, the client’s conversation becomes vulgar. How should the nurse respond to the client’s behavior? 1 Tactfully teasing the client about the use of such vulgarity 2 Restricting the client’s contact with staf f members until the behavior stops Asking the client to limit the use of vulgarity while continuing the conversation A client is not responding to antidepressant medications for treatment of major depression with suicidal ideation. After learning about electroconvulsive therapy (ECT), the client discusses the advantages and disadvantages with the primary nurse. The nurse concludes that the client understands the disadvantages of ECT when the client states that what is one major disadvantage of ECT? 1 The seizures may cause bone fractures. 2 Relief of symptoms requires many weeks of treatment. Memory is impaired after the treatment. The parents of a young adult client visit regularly. After one visit the client becomes very agitated. What should the nurse do to relieve the client’s distress? 1 3 Limit the client’s future contact with the parents. Correct4 Explore the client’s response to the parents’ behavior. A 65-inch (165 cm) tall 15-year-old girl weighing 80 lb (36.3 kg) is admitted to a mental health facility with a diagnosis of anorexia nervosa. The nurse recognizes that her problem most likely is caused by what? Correct1 A desire to contro A nurse concludes that a client has successfully achieved the long-term outcome of mobilizing effective coping responses when the client states the plan to do what when feelings of anxiety begin? Correct1 Perform a relaxation exercise. What should a nurse include in the initial plan of care for a client with the long- standing obsessive-compulsive behavior of handwashing? Determining the purpose of the ritualistic behavior 2 Limiting the time allowed for the ritualistic behavior 3 Suggesting a symptom-substitution technique to refocus the ritualistic behavior Correct4 Developing a routine schedule of activities to reduce the need for the ritualistic behavio What should the nurse do when interacting with an adolescent client with the diagnosis of anorexia nervosa? A client with the diagnosis of bipolar I disorder, manic episode, is hospitalized because he has been stopped by the police several times for reckless driving. The client rarely eats or sleeps and talks constantly. On the second day of hospitalization the client is attempting to organize the other clients in the lounge to form a softball team. What is the most therapeutic nursing intervention? A client is found to have a conversion disorder. What is the typical reaction by the client to the physical symptom? 1 Anger Correct2 Apath A male client with the diagnosis of schizophrenia, paranoid type, often displays overt sexual behavior toward female clients and nurses. What is the nurse’s best response when the client engages in sexually explicit behavior? 1 Refusing to speak with the client until he stops the behavior 2 Sending the client to his room when the behavior is observed 3 Ignoring this behavior until the client is more in control of his responses A client who is experiencing a manic episode of a bipolar I disorder is admitted to a mental health facility. What is a prioritynursing intervention? 2 Arranging for the client to participate in a daily discussion group Correct3 Reducing the number of people interacting with the client during any given day While admitting a young client with anorexia nervosa to the unit, the nurse finds a bottle of assorted pills in the client’s luggage. The client tells the nurse that they are antacids for stomach pains. What is the best initial response by the nurse? "Let’s talk about your drug use." 2 "These pills don’t look like antacids." 3 "Some people take pills to lose weight." Correct4 "Tell me more about these stomach pain A nurse is managing the care of a client with recently diagnosed schizophrenia. Effective therapeutic communication will directly affect which client-focused outcomes? Select all that apply. The client will become capable of part-time employment. The client will effectively express emotional and physical needs. 3 The client will demonstrate wellness reflective of physical potential. The client will demonstrate an understanding of the mental health disorder. Correct5 The client will recognize the issues most impo What is an appropriate way for a nurse to help a client ease anxiety? Avoid unpleasant events. 2 Prolong exposure to fearful situations. 3 Introduce an element of pleasure into fearful situations. Correct4 Help the client acquire skills with which to face stressful events. A client demonstrating manic behavior is elated and sarcastic. The client is constantly cursing and using foul language and has the other clients on the unit terrified. What should the nurse do initially? 1 Demand that the client stop the behavior immediately. Tell the client firmly that the behavior is unacceptable. A client who has been on a psychiatric unit for several weeks continually talks about delusional topics. What response by the nurse is most therapeutic? Asking the client to explain the delusion 2 Allowing the client to maintain the delusion Correct3 Encouraging the client to focus on reality issues The nurse at the mental health clinic is counseling a client with obsessive- compulsive disorder who spends a lot of time each day engaged in handwashing and has trouble keeping appointments on time as a result. What is the most therapeutic initial intervention by the nurse? 2 Encouraging the client to hasten the ritual so appointments can be kept on time 3 Telling the client how angry others become when activities are delayed for handwashing Correct4 Accepting the ritualistic behavior with a matter-of-fact attitude without displaying criticism One day a nurse sits down by a depressed client’s bed and says, "I’ll be spending some time with you today." The client responds, "Go talk to someone else. They all need you more." What is the most therapeutic response by the nurse? 1 "Why do you want me to go?" 2 "I’ll go, but I’ll be back tomorrow." "Don’t you think that you’re important, too?" Correct4 "I’ll be spending the next half hour with you." An older resident in a nursing home who has a diagnosis of dementia hoards leftover food from the meal tray and other seemingly valueless articles and stuffs them into pockets "so the others won’t steal them." What should the nurse plan to do? 1 Remove the resident’s unsafe and soiled articles during the night. Give the resident a small bag in which to place selected personal articles and food. A nurse is caring for a client with generalized anxiety disorder. Which factor should be assessed to best determine the client’s present status? Memory Correct2 Behavior A depressed client says, "I’m no good. I’m better off dead." What is the priority nursing intervention? 1 Responding, "I’ll stay with you until you’re less depressed." Replying, "I think you’re good; you should think about living." Correct3 Alerting the staf f to schedule 24-hour observation of the client A client tells the nurse, "I used to believe that I was God, but now I know that that’s not true." What is the best response by the nurse? "You really believed that?" 2 "Many people have this delusion." Correct3 "This is a sign you are getting better." An older man is widowed suddenly when his wife is killed in an automobile accident. What should the nurse in the emergency department do first to best help the client at this time? 1 Ask a member of the clergy to visit him. Have the primary healthcare provider prescribe a sedative for him. 3 Refer him to a support group that meets near his home. Correct4 Assure him that everything possible was done for his wife. The nurse finds a client with schizophrenia lying under a bench in the hall. The client says, "God told me to lie here." What is the best response by the nurse? Correct1 "I didn’t hear anyone talking; come with me to your room." A nurse is working in a daycare center with clients who have cognitive impairments. What does the nurse expect of a client in the middle stages of dementia? Ability to recall events from the past A 6-year-old child recently started school but has been refusing to go for the past 3 weeks. What would be an appropriate nursing intervention for this child? 1 Explain that school is a place to have fun. 2 Delay the return to school for several months. 3 Enroll the child in a special education program. Develop a behavior modification program with the child. A nurse has been caring for a suicidal client for 3 weeks on an inpatient unit. One morning the client greets the nurse cheerfully and states, "Everything is looking up. I’m not going to have problems for very long." What does the client’s behavior and statement indicate? Increased risk of suicide 2 Increased level of anxiety A nurse identifies the establishment of trust as a major nursing goal for a depressed client. How can this goal best be accomplished? 1 By spending a day with the client 2 By asking the client at least one question daily 3 hat is the best nursing intervention to encourage a socially withdrawn client to talk? Correct1 Focusing on nonthreatening subjects 2 Trying to get the client to A client with a diagnosis of panic disorder who had a panic attack on the previous day says to the nurse, "That was a terrible feeling I had yesterday. I’m so afraid to talk about it." What is the most therapeutic response by the nurse? 1 "Okay; we don’t have to talk about it." 2 "Why don’t you want to talk about it?" "What were you doing yesterday when you first noticed the feeling A housekeeping staff member in a mental health unit reports to the nurse that food was found hidden in a client’s room. Knowing that the client was admitted with a fluid and electrolyte imbalance because of anorexia nervosa, what should the nurse ask housekeeping personnel to do? 1 Point this out to the client and remove the food. Correct2 Report it to the nursing staf f if it happens again A nurse is caring for an adolescent with the diagnosis of anorexia nervosa. The plan of care should include helping the client do what? Plan nutritious meals. 2 Change attitudes about nutrition. 3 Correct4 Recognize how the need to control influences behavior. What should the nurse do when implementing a tertiary preventive program for cognitively impaired individuals? Correct1 Teach children how to feed themselves A client with schizophrenia uses the word "worriation" when talking with the nurse. How should the nurse respond? 1 By correcting the pronunciation of the word Correct2 By asking for clarification of the word’s meaning What should a nurse do when caring for a client whose behavior is characterized by pathologic suspicion? Protect the client from environmental stress. Correct2 Help the client feel accepted by the staf f on the uni nch is being served, and the clients must walk to the dining room. The nurse finds one client sitting alone with the head slightly tilted as if listening to something. How should the nurse respond? 1 "I know you’re busy, but it’s lunchtime." "Are the voices bothering you again?" 3 "Get going; you don’t want to miss lunchtime The multidisciplinary team decides to use a behavior modification approach for a young client with anorexia nervosa. Which planned nursing intervention is an appropriate approach to use with this client? 1 Correct3 Restricting the client to the room until a 2-lb (0.9-kg) weight gain is achieved An 84-year-old widow with dementia who had been living with her daughter before hospitalization is being discharged with a referral to the visiting nurse. When the nurse visits, the client is in bed sleeping at 10:00 am. Her daughter states that she gives her mother sleeping pills to stop her wandering at night. How should the nurse respond? Correct1 Explore hiring a home health aide to stay with the client a The nurse admits an adolescent to the psychiatric unit with the diagnosis of anorexia nervosa. What is the primary gain a client with anorexia achieves from this disorder? Correct1 Reduction of anxiety through control over food A client is hospitalized for a bipolar mood disorder, manic episode. The client is hyperactive and obnoxious, calls the nurse names, is sarcastic to the staff, and taps the nurse playfully on the buttocks. What is the most important action of the nurse? 1 Spend extra time with the client. Place the client alone in a quiet room. 3 Disregard the client’s acting-out behavior. Correct4 Assess personal feelings toward the clie Electroconvulsive therapy (ECT) is a mode of treatment that is used primarily to treat what? Correct1 Clinical depressi A client with schizophrenia who has auditory hallucinations is withdrawn and apathetic. What should the nurse say to involve this client in an activity? 1 "You’ll get a reward if you go to the gym." Correct2 "Would you like to participate in the group walk toda fter an automobile accident a person is arrested for driving while intoxicated and is admitted to the hospital. When the client becomes angry and blames the family for personal problems, the nurse can be most therapeutic by using which statement? 1 "You know that you are to blame for your alcohol abuse." "You need help now or you’re just going to get even sicker." 3 "I’ll talk to your family about their behavior if you want me to." Correct4 y the nurse and a young adult client sit together and draw. The client draws a face with horns and says, "This is me. I’m a devil." What is the best response by the nurse? 1 "I don’t see a devil; why do you see a devil?" 2 "Let’s go to the mirror to see what you look like." Correct3 "When I look at you I see a person, not a devil." A nurse is planning health teaching for a 14-year-old girl hospitalized with the diagnosis of anorexia nervosa. What does the nurse assume is likely true of the client? 1 Correct3 Has minimal awareness that reduced caloric intak A nurse is discharging a client from the mental health unit who has been treated for major depression. Which statement is most therapeutic at this time? 1 "I’m going to miss you; we’ve become good friends." "I know that you’re going to be all right when you go home." Correct3 "Call the contact number we gave you if y An acutely ill client with the diagnosis of schizophrenia has just been admitted to the mental health unit. What is the most therapeutic initial nursing intervention? Correct1 Spending time with the A nurse is developing a care plan for a client with obsessive-compulsive behavior disorder. Which nursing intervention will most likely increase the client’s anxiety? Helping the client understand the nature of the anxiety Correct2 Limiting the client’s ritualistic acts to three ti n autistic toddler is sitting in a corner, rocking and spinning a top. How can the nurse be most therapeutic when approaching this toddler? 1 2 Correct4 Sitting with the toddler while watching the spinning to What is the best nursing intervention when the language of a client in the manic phase of a bipolar disorder becomes vulgar and profane? 1 Stating, "We don’t like that kind of talk around here." 2 Ignoring it because the client is using it to gain attention Recognizing that the behavior is part of the illness but setting limit A nurse observes a client with developmental delays eating soft food without utensils. What is the best nursing intervention? Correct1 Giving the client a spoon and suggesting that it be used 2 Saying jokingly, "Well, I guess fingers were made b A nurse is caring for a newly admitted client with anorexia nervosa. What is the prioritytreatment for the client at this time? 1 Medications to reduce anxiety 2 Family psychotherapy sessions Separation from family members Correct4 Correction of electrolyte During a nursing team conference, a mental health worker suggests that a client with schizophrenia, paranoid type, be assigned to group therapy. What should the nurse manager explain about this type of therapy for this client? 1 Individuals with this disorder respond well to small therapeutic groups. Therapeutic group work tends to be threatening to individuals who are suspicious. 3 Compliance with unit rules A client with paranoid schizophrenia tells the nurse, "My neighbors are spying on me because they want to rob me and take money." While hospitalized, the client complains of being poisoned by the food and of being given the wrong medication. The nurse evaluates the client’s response to medications and therapy. Which assessment finding leads the nurse to conclude that the client’s reality testing has improved? Correct1 The client eats the food proA 3-year-old client has been admitted to the pediatric unit for dehydration resulting from nausea and vomiting. The parents tell the nurse the child has autism and resists being held, acts as if deaf, frequently mimics words or phrases, and is not toilet trained. What is most important for the nurse to do when planning care for this child? 1 Provide a structured routine for the child to follow while in the hospital Correct2 How can the nurse best minimize psychologic stress in an anxious client who has been admitted to the psychiatric unit? 1 Explain in detail the therapies being used. Correct2 Learn what is of particular importance to the clien hen interacting with an adolescent client with the diagnosis of anorexia nervosa, what is most important for the nurse to do? 1 Show empathy. 2 Maintain control. Correct3 [Show More]

Last updated: 1 year ago

Preview 1 out of 114 pages

Reviews( 0 )

Recommended For You

 *NURSING> EXAM REVIEW > RNSG 2201 _ EXAM 5 RNSG 2201 | Download To Score An A+ | Answers Correctly Marked 100% (All)

preview
RNSG 2201 _ EXAM 5 RNSG 2201 | Download To Score An A+ | Answers Correctly Marked 100%

EXAM # 5 RNSG 2201 CH. 23, 24, 25, 26 Chapter 23: The Child with Fluid and Electrolyte Imbalance MULTIPLE CHOICE 1. What substance is released from the posterior pituitary gland and promote...

By A+ Solutions , Uploaded: Mar 16, 2021

$20.5

 *NURSING> EXAM REVIEW > NURS 6501 Midterm Exam Review Guide (Weeks 1-6. Compilation in 99 Pages) (All)

preview
NURS 6501 Midterm Exam Review Guide (Weeks 1-6. Compilation in 99 Pages)

Nurs 6501 Midterm Exam Review Guide (Weeks 1-6) Cellular Processes and the Genetic Environment 1. Describe cellular processes and alterations within cellular processes. 2. What is the impact of t...

By SuperSolutions© , Uploaded: Nov 24, 2020

$15

 *NURSING> EXAM REVIEW > NUR2063 / NUR 2063 Essentials of Pathophysiology Exam Review Latest Update Rasmussen College (All)

preview
NUR2063 / NUR 2063 Essentials of Pathophysiology Exam Review Latest Update Rasmussen College

NUR 2063 Essentials of Pathophysiology Exam Review 1. A potentially lethal condition in which there is an acute elevation of circulating thyroid hormones is called _______________ - ANS: Thy...

By quiz_bit , Uploaded: Oct 28, 2020

$12

 *NURSING> EXAM REVIEW > Multidimensional Care (MDC) 1 MDC EXAM 2 REVIEW -. (Rasmussen College) (All)

preview
Multidimensional Care (MDC) 1 MDC EXAM 2 REVIEW -. (Rasmussen College)

MDC EXAM 2 REVIEW 11:19:19 1. The effects of Immobility a) Interventions that improve flexibility • P.R.E.P.(Perform passive ROM, Reposition Q2HR, Encourage independent activity as much as possible...

By SuperSolutions© , Uploaded: Dec 11, 2020

$11

 *NURSING> EXAM REVIEW > NUR 2356 / NUR2356 Multidimensional Care I (MDC I) Exam 2 Review. Rasmusssen College (All)

preview
NUR 2356 / NUR2356 Multidimensional Care I (MDC I) Exam 2 Review. Rasmusssen College

1. What is a function of the musculoskeletal system? -Assist with movement 2. As a nurse you know that during aging, a normal musculoskeletal change would be? -A patient that came to the cli...

By nurse_steph , Uploaded: Nov 15, 2020

$11

 *NURSING> EXAM REVIEW > NR566 / NR 566 Advanced Pharmacology for Care of the Family Final Exam Week 5 Review (Graded A) Chamberlain College of Nursing (All)

preview
NR566 / NR 566 Advanced Pharmacology for Care of the Family Final Exam Week 5 Review (Graded A) Chamberlain College of Nursing

NR-566 Advanced Pharmacology for Care of the Family Final Exam Week 5 Review 1. What therapy and its dose is started within 48 hours of a TIA or ischemic stroke? - 2. Causes of sickling / S...

By quiz_bit , Uploaded: Oct 08, 2020

$9

 *NURSING> EXAM REVIEW > NUR 2063 / NUR2063 Essentials of Pathophysiology Final Exam Review Modules 1 & 2 & 3 Latest Update Rasmussen (All)

preview
NUR 2063 / NUR2063 Essentials of Pathophysiology Final Exam Review Modules 1 & 2 & 3 Latest Update Rasmussen

Modules 1 & 2 & 3 Essentials of Pathophysiology Final Exam Review

By quiz_bit , Uploaded: Nov 02, 2020

$15

 *NURSING> EXAM REVIEW > NURS 6501 / NURS6501 Advanced Pathophysiology - Module 8 Knowledge Check (Latest Update) Walden University (All)

preview
NURS 6501 / NURS6501 Advanced Pathophysiology - Module 8 Knowledge Check (Latest Update) Walden University

Review Test Submission: Module 8 Knowledge Check

By quiz_bit , Uploaded: Jan 13, 2021

$9.5

 *NURSING> EXAM REVIEW > NR566 / NR 566 Advanced Pharmacology for Care of the Family Week 3 Quiz Study Guide . (Graded A) Chamberlain College of Nursing (All)

preview
NR566 / NR 566 Advanced Pharmacology for Care of the Family Week 3 Quiz Study Guide . (Graded A) Chamberlain College of Nursing

NR-566 Advanced Pharmacology for Care of the Family Week 3 Quiz Study Guide If a patient is taking an ACEI and develops a persistent dry cough, what should the provider do? Which cl...

By quiz_bit , Uploaded: Oct 08, 2020

$15

 *NURSING> EXAM REVIEW > NR566 / NR 566 Advanced Pharmacology for Care of the Family Midterm Exam Review . (Graded A) Chamberlain College of Nursing (All)

preview
NR566 / NR 566 Advanced Pharmacology for Care of the Family Midterm Exam Review . (Graded A) Chamberlain College of Nursing

NR-566 Advanced Pharmacology for Care of the Family NR566 Midterm Differentiate between primary and secondary hyperthyroidism Primary is the term used when the pathology is within the thyro...

By quiz_bit , Uploaded: Nov 21, 2020

$14

$20.00

Add to cart

Instant download

Can't find what you want? Try our AI powered Search

OR

GET ASSIGNMENT HELP
143
0

Document information


Connected school, study & course



About the document


Uploaded On

Mar 11, 2021

Number of pages

114

Written in

Seller


seller-icon
A+ Solutions

Member since 3 years

164 Documents Sold


Additional information

This document has been written for:

Uploaded

Mar 11, 2021

Downloads

 0

Views

 143

Document Keyword Tags

THE BEST STUDY GUIDES

Avoid resits and achieve higher grades with the best study guides, textbook notes, and class notes written by your fellow students

custom preview

Avoid examination resits

Your fellow students know the appropriate material to use to deliver high quality content. With this great service and assistance from fellow students, you can become well prepared and avoid having to resits exams.

custom preview

Get the best grades

Your fellow student knows the best materials to research on and use. This guarantee you the best grades in your examination. Your fellow students use high quality materials, textbooks and notes to ensure high quality

custom preview

Earn from your notes

Get paid by selling your notes and study materials to other students. Earn alot of cash and help other students in study by providing them with appropriate and high quality study materials.


$20.00

WHAT STUDENTS SAY ABOUT US


What is Browsegrades

In Browsegrades, a student can earn by offering help to other student. Students can help other students with materials by upploading their notes and earn money.

We are here to help

We're available through e-mail, Twitter, Facebook, and live chat.
 FAQ
 Questions? Leave a message!

Follow us on
 Twitter

Copyright © Browsegrades · High quality services·